(3 intermediate revisions by 2 users not shown)
Line 1: Line 1:
[[Category:MA453Spring2009Walther]]Let <math>\scriptstyle p</math> be a prime. Describe <math>\scriptstyle U(p^n)</math> and find the order of this group. Using your guess from HW3 in Chapter 3 now predict <math>\scriptstyle\mid U(750)\mid</math>.
+
=[[Week_5|Week 5 HW]], Chapter 6, Problem 7, [[MA453]], Spring 2008, [[user:walther|Prof. Walther]]=
 +
==Problem Statement==
  
 +
Let <math>\scriptstyle p</math> be a prime. Describe <math>\scriptstyle U(p^n)</math> and find the order of this group. Using your guess from HW3 in Chapter 3 now predict <math>\scriptstyle\mid U(750)\mid</math>.
 
----
 
----
 +
==Discussion==
 
To gain some insight into this problem, lets compute <math>\scriptstyle U(p^n)</math> for  a few values of <math>\scriptstyle n</math>. Let <math>\scriptstyle p</math> be 3, for simplicity purposes.
 
To gain some insight into this problem, lets compute <math>\scriptstyle U(p^n)</math> for  a few values of <math>\scriptstyle n</math>. Let <math>\scriptstyle p</math> be 3, for simplicity purposes.
  
Line 75: Line 78:
 
Dane, thanks for the kind words, I'm just glad to help out (and call me crazy, but challenging proofs are kinda fun lol).
 
Dane, thanks for the kind words, I'm just glad to help out (and call me crazy, but challenging proofs are kinda fun lol).
 
:--[[User:Narupley|Nick Rupley]] 02:51, 12 February 2009 (UTC)
 
:--[[User:Narupley|Nick Rupley]] 02:51, 12 February 2009 (UTC)
 +
 +
I don't understand how you came about the formula.  Did you try test cases and make a general formula from them or was it more intuition.  I just couldn't see the relationship.
 +
-- [[User:rpkersey|Ross Kersey]]
 +
 +
----
 +
Well, I started out with <math>\scriptstyle U(3^1)\ =\ \{1,2\}</math> and <math>\scriptstyle U(3^2)\ =\ \{1,2,4,5,7,8\}</math>, and noted that when you multiply the power of <math>\scriptstyle p</math> (in this case <math>\scriptstyle3^n</math>) by another <math>\scriptstyle p</math>, you are expanding the cardinality of the set <math>\scriptstyle\{1,2,\ldots,p^n\}</math> by <math>\scriptstyle p</math>. So, the number of multiples of <math>\scriptstyle p</math> that appear in the expanded set is going to be <math>\scriptstyle p</math> times more than the original set. The set of <math>\scriptstyle\{1,2,\ldots,p\}</math> contains only one multiple of <math>\scriptstyle p</math> (<math>\scriptstyle p</math> itself), so it would make sense that <math>\scriptstyle\{1,2,\ldots,p^2\}</math> contains <math>\scriptstyle p</math> multiples of <math>\scriptstyle p</math>, and so on. So to answer your question, I suppose it was a bit of intuition which came from a couple of test cases.
 +
:--[[User:Narupley|Nick Rupley]] 03:13, 13 February 2009 (UTC)
 +
----
 +
[[Week_5|Back to Week 5 Homework]]
 +
 +
[[MA453_(WaltherSpring2009)|Back to MA453 Spring 2009 Prof. Walther]]
 +
 +
[[Category:MA453Spring2009Walther]]

Latest revision as of 17:29, 22 October 2010

Week 5 HW, Chapter 6, Problem 7, MA453, Spring 2008, Prof. Walther

Problem Statement

Let $ \scriptstyle p $ be a prime. Describe $ \scriptstyle U(p^n) $ and find the order of this group. Using your guess from HW3 in Chapter 3 now predict $ \scriptstyle\mid U(750)\mid $.


Discussion

To gain some insight into this problem, lets compute $ \scriptstyle U(p^n) $ for a few values of $ \scriptstyle n $. Let $ \scriptstyle p $ be 3, for simplicity purposes.

$ \scriptstyle U(3^1)\ =\ \{1,2\} $

$ \scriptstyle U(3^2)\ =\ \{1,2,4,5,7,8\} $

$ \scriptstyle U(3^3)\ =\ \{1,2,4,5,7,8,10,11,13,14,16,17,19,20,22,23,25,26\} $

Notice how the only gaps in the sequence of numbers $ \scriptstyle\{1,2,\ldots,p^n-1,p^n\} $ are those numbers divisible by $ \scriptstyle p $. Since $ \scriptstyle p $ is prime, we should expect this. The question then boils down to exactly how many of these gaps there are. In $ \scriptstyle U(3) $, there is one gap (including $ \scriptstyle p^n $). In $ \scriptstyle U(9) $ there are three, and in $ \scriptstyle U(27) $, there are nine. It would seem that the number of such gaps is equal to the prime power of the previous unit group. Then, the order of the group $ \scriptstyle U(p^n) $ would be $ \scriptstyle p^n-p^{n-1} $. Let's try to prove this.

Observe that for $ \scriptstyle n=1 $, $ \scriptstyle U(p^n)\ =\ U(p)\ =\ \{1,2,\ldots,p-2,p-1\} $, with no gaps (save for $ \scriptstyle p $ itself), such that $ \scriptstyle\mid U(p^n)\mid\ =\ p-1\ =\ p^1-p^0\ =\ p^n-p^{n-1}\checkmark $

Now assume that for an arbitrary but finite $ \scriptstyle n $, $ \scriptstyle\mid U(p^n)\mid\ =\ p^n-p^{n-1} $.

We now want to think of what happens when we move from $ \scriptstyle U(p^n) $ to $ \scriptstyle U(p^{n+1}) $. We now have the sequence $ \scriptstyle\{1,2,\ldots,p^{n+1}-1\} $, minus all the gaps due to the multiples of $ \scriptstyle p $. For now we can disregard all the numbers from $ \scriptstyle1 $ to $ \scriptstyle p^n $, since we already know how many gaps are in there.

Following $ \scriptstyle p^n $, we have $ \scriptstyle p^n+1,p^n+2,\ldots,p^n+p-2,p^n+p-1 $. The reason we have the $ \scriptstyle+p $ factor is because we want to approach the next multiple of $ \scriptstyle p $ without actually including it (obviously since it can't be included in the unit group of $ \scriptstyle p^{n+1} $). In similar fashion, the next "section" would be $ \scriptstyle p^n+p+1,p^n+p+2,\ldots,p^n+2p-2,p^n+2p-1 $.

This process of counting the elements in between the multiples of $ \scriptstyle p $ continues until we reach the end: $ \scriptstyle p^n+(p^n-p^{n-1}-1)\cdot p+1,\ p^n+(p^n-p^{n-1}-1)\cdot p+2,\ \ldots,\ p^n+(p^n-p^{n-1})\cdot p-2,\ p^n+(p^n-p^{n-1})\cdot p-1 $.

So, why is the last multiple of $ \scriptstyle p $ we are approaching $ \scriptstyle p^n+(p^n-p^{n-1})\cdot p $? Well, this is because $ \scriptstyle p^n+(p^n-p^{n-1})\cdot p\ =\ p^n+p^{n+1}-p^n\ =\ p^{n+1} $. Obviously the last multiple of $ \scriptstyle p $ we want to approach is $ \scriptstyle p^{n+1} $, since we are dealing with $ \scriptstyle U(p^{n+1}) $.

To recap, we have:

$ \scriptstyle U(p^{n+1})\ =\ \textstyle\{\scriptstyle\overbrace{\scriptstyle1,2,\ldots,p^n-2,p^n-1}^{p^n-p^{n-1}}\textstyle,\scriptstyle\ \ \overbrace{\scriptstyle p^n+1,p^n+2,\ldots,p^n+p-2,p^n+p-1}^{p-1}\textstyle, $

$ \scriptstyle\overbrace{\scriptstyle p^n+p+1,p^n+p+2,\ldots,p^n+2p-2,p^n+2p-1}^{p-1}\textstyle,\ \cdots, $

$ \scriptstyle\overbrace{\scriptstyle p^n+(p^n-p^{n-1}-2)\cdot p+1,\ p^n+(p^n-p^{n-1}-2)\cdot p+2,\ \ldots,\ p^n+(p^n-p^{n-1}-1)\cdot p-2,\ p^n+(p^n-p^{n-1}-1)\cdot p-1}^{p-1}\textstyle, $

$ \scriptstyle\overbrace{\scriptstyle p^n+(p^n-p^{n-1}-1)\cdot p+1,\ p^n+(p^n-p^{n-1}-1)\cdot p+2,\ \ldots,\ p^n+(p^n-p^{n-1})\cdot p-2,\ p^n+(p^n-p^{n-1})\cdot p-1\ =\ p^{n+1}-1}^{p-1}\textstyle\} $

Each section of elements (denoted by the overbraces) after $ \scriptstyle p^n $ contains exactly $ \scriptstyle p-1 $ elements, because we are counting the number of elements in between each multiple of $ \scriptstyle p $. We have $ \scriptstyle p^n-p^{n-1} $ such sections of length $ \scriptstyle p-1 $. This is because in each section, recall that we are "approaching" the next multiple of $ \scriptstyle p $. In the second overbrace we approach $ \scriptstyle p^n+1\cdot p $, in the third overbrace $ \scriptstyle p^n+2\cdot p $, and so on until in the last section we approach $ \scriptstyle p^n+(p^n-p^{n-1})\cdot p $.

Now, we can finally count the number of elements in $ \scriptstyle U(p^{n+1}) $. The first section has $ \scriptstyle p^n-p^{n-1} $ elements; this comes from the inductive assumption. We have also just shown that the remaining $ \scriptstyle p^n-p^{n-1} $ sections each have $ \scriptstyle p-1 $ elements, for a total of $ \scriptstyle (p^n-p^{n-1})*(p-1)\ =\ p^{n+1}-2p^n+p^{n-1} $ elements. Adding this to the number of elements from the first section, we have a grand total of:

$ \scriptstyle p^n-p^{n-1}+p^{n+1}-2p^n+p^{n-1}\ =\ p^{n+1}-p^n $ elements.

It follows that $ \scriptstyle\mid U(p^{n+1})\mid\ =\ p^{n+1}-p^n $.

We have proved that $ \textstyle(\scriptstyle\mid U(p^n)\mid\ =\ p^n-p^{n-1}\textstyle)\ \rightarrow\ (\scriptstyle\mid U(p^{n+1})\mid\ =\ p^{n+1}-p^n\textstyle) $, and have shown that $ \scriptstyle\mid U(p^n)\mid\ =\ p^n-p^{n-1} $ for $ \scriptstyle n=1 $, so by induction, $ \scriptstyle\mid U(p^n)\mid\ =\ p^n-p^{n-1} $ for all $ \scriptstyle n\in\mathbb{N} $. $ \scriptstyle\Box $


To predict $ \scriptstyle\mid U(750)\mid $, note that $ \scriptstyle750\ =\ 2*3*5*5*5\ =\ 6*5^3 $. $ \scriptstyle gcd(6,5^3)\ =\ 1 $, so by our conjecture in the Chapter 3 homework, $ \scriptstyle\mid U(750)\mid\ =\ \mid U(6)\mid*\mid U(5^3)\mid $. $ \scriptstyle U(6)\ =\ \{1,5\} $, so $ \scriptstyle\mid U(6)\mid\ =\ 2 $. We just proved that $ \scriptstyle\mid U(p^n)\mid\ =\ p^n-p^{n-1} $, so $ \scriptstyle\mid U(5^3)\mid\ =\ 5^3-5^2\ =\ 125-25\ =\ 100 $.

Thus, $ \scriptstyle\mid U(750)\mid\ =\ 2*100\ =\ 200 $.

--Nick Rupley 03:03, 9 February 2009 (UTC)

well done.



I have a question of clarification. Should the above be:

To recap, we have:

$ \scriptstyle U(p^{n+1})\ =\ \textstyle\{\scriptstyle\overbrace{\scriptstyle1,2,\ldots,p^n-2,p^n-1}^{p^n-p^{n-1}}\textstyle,\scriptstyle\ \ \overbrace{\scriptstyle p^n+1,p^n+2,\ldots,p^n+p-2,p^n+p-1}^{p-1}\textstyle, $

?? In the original it's said the first overbrace has $ \scriptstyle p^n-p^{n+1} $ but I thought it contained $ \scriptstyle p^n-p^{n-1} $. (Especially since $ \scriptstyle p^n-p^{n+1} $ would be a negative number...) I'm not trying to be nit-picky, just trying to contribute for credit  :) --Bcaulkin 00:27, 12 February 2009 (UTC)


Man this stuff is like a square peg in a round hole. Great work Nick. I'm still trying to understand the majority of it, but the result is quite cool. I think you should run a recitation.

-- Dane DeSutter


Bcaulkin (sorry I don't know your name ^^;;), you're right, it should be $ \scriptstyle p^n-p^{n-1} $, my mistake. It's fixed now though.

Dane, thanks for the kind words, I'm just glad to help out (and call me crazy, but challenging proofs are kinda fun lol).

--Nick Rupley 02:51, 12 February 2009 (UTC)

I don't understand how you came about the formula. Did you try test cases and make a general formula from them or was it more intuition. I just couldn't see the relationship. -- Ross Kersey


Well, I started out with $ \scriptstyle U(3^1)\ =\ \{1,2\} $ and $ \scriptstyle U(3^2)\ =\ \{1,2,4,5,7,8\} $, and noted that when you multiply the power of $ \scriptstyle p $ (in this case $ \scriptstyle3^n $) by another $ \scriptstyle p $, you are expanding the cardinality of the set $ \scriptstyle\{1,2,\ldots,p^n\} $ by $ \scriptstyle p $. So, the number of multiples of $ \scriptstyle p $ that appear in the expanded set is going to be $ \scriptstyle p $ times more than the original set. The set of $ \scriptstyle\{1,2,\ldots,p\} $ contains only one multiple of $ \scriptstyle p $ ($ \scriptstyle p $ itself), so it would make sense that $ \scriptstyle\{1,2,\ldots,p^2\} $ contains $ \scriptstyle p $ multiples of $ \scriptstyle p $, and so on. So to answer your question, I suppose it was a bit of intuition which came from a couple of test cases.

--Nick Rupley 03:13, 13 February 2009 (UTC)

Back to Week 5 Homework

Back to MA453 Spring 2009 Prof. Walther

Alumni Liaison

has a message for current ECE438 students.

Sean Hu, ECE PhD 2009